Radiology Flashcards
A 56-year-old man presents to the emergency room
with 1 week of altered mental status. His medical
history is significant for a glioblastoma treated with
resection followed by temozolomide therapy and
whole brain radiation 1 year ago. An MRI is performed, and contrast-enhanced, diffusion-weighted,
and apparent diffusion coefficient sequences are
shown in these images. Perfusion maps (not shown)
demonstrate decreased relative cerebral blood volume. What is the likely cause of his new symptoms?
A. Radiation necrosis
B. Recurrent glioblastoma
C. Secondary tumor caused by chemotherapy
regimen
D. Encephalomalacia from tumor resection
A. Radiation necrosis
Radiation necrosis typically presents with higher apparent diffusion coefficient (ADC) values compared with tumor recurrence, which tends to show restricted diffusion and therefore lower signal in the solid enhancing components. On perfusion MRI,
radiation necrosis demonstrates decreased relative cerebral blood volume (rCBV) in contrast to highgrade tumors. Finally, this patient is 1 year from his radiation therapy, which is a peak time for radiation necrosis to appear (usually between 12 and 24 months). (B) A recurrent glioblastoma would tend to have a low ADC signal and increased perfusion on rCBV maps. (C) Secondary malignancies following temozolomide (an alkylating agent) have been reported but are a rare occurrence. These malignancies usually are hematologic. Glioblastoma would be much more likely than a secondary malignancy, given its aggressive nature and expected eventual recurrence. (D) Encephalomalacia would be apparent by parenchymal volume loss and should not
show nodular enhancement or restricted diffusion. Some (usually nonnodular) enhancement may be present around the resection cavity due to granulation tissue
The lesion shown in these images depicts a(n):
A. Optic nerve glioma
B. Meningioma
C. Chordoma
D. Esthesioneuroblastoma
D. Esthesioneuroblastoma
Esthesioneuroblastomas (olfactory neuroblastomas) are rare, malignant tumors of the superior nasal cavity and anterior skull base. Treatment strategies vary widely and include surgery, radiotherapy, and/or chemotherapy. The ideal treatment
modality has yet to be determined. Esthesioneuroblastomas tend to exert mass effect on the orbits, optic nerves, and optic chiasm, and may result in proptosis. They can occur in the frontal sinus and have variable intracranial extension. They homogeneously enhance on contrasted T1 sequences and
appear moderately hyperintense on T2 sequences. (A) An optic nerve glioma would infiltrate and expand the optic nerve. As seen in the second image, there is stretching of the optic nerve due to mass effect and proptosis, but the tumor itself does not involve the optic apparatus. (B) Anterior skull base meningiomas also demonstrate avid enhancement and occasionally can extend into the olfactory recess. They may present with a dural tail
and hyperostosis; however, the epicenter of the mass in this case is in the sinonasal cavity without a significant intracranial dural component. This makes a meningioma unlikely. Esthesioneuroblastomas often do extend intracranially, in which case a “waist” at the level of the cribriform plate and cysts at the brain–tumor interface are strongly suggestive of such diagnosis. (C) Skull base chordomas usually are located in the anterior clivus but
rarely may extend to the nasal cavity. Primary
chordomas in the nasal cavity and maxilla without clival involvement are extremely rare. Chordomas usually demonstrate more heterogeneous enhancement than esthesioneuroblastomas on contrasted
T1 sequences and are very hyperintense on T2
sequences.
What likely is associated with the imaging findings
on the MR susceptibility-weighted imaging (SWI)
sequence shown in these images?
A. Shearing injury from rotational acceleration
B. β-amyloid peptide deposits
C. Mutations in the CCM1 gene
D. Long bone fractures
D. Long bone fractures
Long bone fractures can result in fat embolism,
which appears as tiny foci of susceptibility artifact
that preferentially may be located at the gray matter–white matter interface or diffusely distributed
as in this case. Fat embolism may be accompanied
by scattered foci of restricted diffusion. (A) Diffuse
axonal injury is due to shearing forces from rotational acceleration. Areas of hemorrhage can be
detected on susceptibility-weighted imaging (SWI)
sequences and tend to be located in the gray
matter–white matter junction, corpus callosum,
and brainstem. Linear SWI signal loss often can be
seen along the white matter tracts. (B) β-amyloid
peptide deposits are related to cerebral amyloid
angiopathy (CAA). SWI may show multiple foci
of signal loss located peripherally in a cortical/
subcortical distribution rather than diffusely as
in this case. Patients also can have superficial siderosis and present with subarachnoid hemorrhage.
(C) Mutations in the CCM1, CCM2, and CCM3 genes
can be seen in familial cavernous malformation
syndromes. Lesions usually are more randomly
distributed and not as evenly sized as in the current case.
A 22-year-old man without a significant medical
history presents with progressive midthoracic pain.
An MRI examination of the spine is shown in these
images. What is the most likely diagnosis?
A. Astrocytoma
B. Ependymoma
C. Metastasis
D. Tumefactive demyelination
A. Astrocytoma
Astrocytomas in the spinal cord are most common in children but also may occur in adults. The
majority of them are histologically low grade and
slow growing, and may result in bone remodeling.
Half of astrocytomas are of the pilocytic subtype
and are relatively well defined, whereas the
remainder are infiltrative. They commonly present
as expansile masses with variable degrees of illdefined enhancement, although some tumors may
not enhance. (B) Ependymomas are more common in adults. Compared with astrocytomas, their
enhancement more frequently is well defined, and
they more commonly are associated with cystic
changes or hemorrhage. They are located more
centrally, as they arise from the ependyma, but
such distinction becomes difficult in larger tumors.
(C) Metastases to the cervical spine typically are
more focal than and not as expansile as the demonstrated lesion. It would be highly unusual for a
patient of this age to present with a spinal cord
metastasis without a known primary malignancy.
(D) Tumefactive demyelination is more common
in the brain. Demyelinating lesions may be seen in
the spinal cord and may be related to acute disseminated encephalomyelitis (more common in
the cervical spine), neuromyelitis optica (which
may result in longitudinally extensive transverse
myelitis), and multiple sclerosis (usually with small
lesions centered in the peripheral white matter
and only rarely extending over long segments when
confluent). The appearance of the lesion in question is too expansile and is not compatible with a
demyelinating or inflammatory process.
A neonate underwent an MRI of his brain, shown
in this image. What may be an associated finding?
A. Interhemispheric cysts
B. Collapse of ventricular atria and occipital horns
C. Low-riding third ventricle
D. Curvilinear pericallosal lipomas
A. Interhemispheric cysts
The image accompanying the question shows
dysgenesis of the corpus callosum with incomplete
formation of the cingulate gyrus. Dorsal or occasionally anterior interhemispheric cysts are a common finding. (B) Colpocephaly (dilatation of the
atria and occipital horns of the lateral ventricles) is
a characteristic secondary finding in dysgenesis of
the corpus callosum. (C) Patients with dysgenesis
of the corpus callosum can have a “high-riding”
third ventricle. Additional findings include bundles of Probst (white matter that failed to cross the
midline) and incomplete rotation of the hippocampi. (D) Tubulonodular lipomas have a significantly increased incidence of associated anomalies
compared with curvilinear ones.
What is a characteristic of the lesion depicted in
the MRI study shown in these images?
A. Results from premature disjunction of the
cutaneous ectoderm from the neuroectoderm
during neurulation
B. Infiltrative hypercellular lesion with variable
degrees of mitosis/atypia
C. Results from clonal transformation of cells of
B-cell origin
D. May be associated with endolymphatic sac
tumors, ren
A. Results from premature disjunction of the
cutaneous ectoderm from the neuroectoderm
during neurulation
Postcontrast sagittal T1 and short tau inversion
recovery (STIR) images show a large lobulated
mass that appears to be intradural and at least partially intramedullary. Although most of the mass
is hyperintense on T1, these areas fully suppress
on STIR images, indicating that this signal is related
to fat rather than contrast enhancement. Spinal
lipomas follow fat signal on all sequences, and subcutaneous fat can be used as an internal control.
Embryologically, these lesions result from premature disjunction of the cutaneous ectoderm from
the neuroectoderm during neurulation, whereby
the open neural tube becomes exposed to the
ingrowth of mesodermal tissues. Patients may
experience slowly progressing paresis, spasticity,
or sensory loss depending on the extent and location of the lesion. (B) An infiltrative hypercellular
lesion with variable degrees of mitosis/atypia would
describe a neoplastic process such as an astrocytoma, which may or may not show enhancement.
(C) Clonal transformation of cells of B-cell origin
would result in lymphoma (e.g., Hodgkin). These
lesions are rare in the spinal cord and usually
enhance. (D) Hemangioblastomas are very vascular and avidly enhance. They can occur in the spinal cord or leptomeninges. They may be associated
with endolymphatic sac tumors, renal cell carcinomas, retinal angiomas, pheochromocytomas, and
cystic lesions.
A 30-year-old woman with a history of recurrent
genital and oral aphthae and erythema nodosum
underwent an MRI of the brain. What is a likely
imaging finding in this patient?
A. Enhancing lesion involving the brainstem
B. Fluid-attentuated inversion recovery (FLAIR)
hyperintense lesion sparing red nuclei and
substantia nigra
C. Lesions with a leading edge of restricted
diffusion
D. Lesions involving the pulvinar and dorsomedial thalamic nuclei
E. Lesions with an incomplete rim of enhancement
A. Enhancing lesion involving the brainstem
Manifestations of Behçet disease in the central
nervous system are varied. The brainstem is the
most commonly affected, and lesions can be focal
or multifocal. There usually is associated edema
and contrast enhancement, particularly in the acute
phase. Patients also can present with meningoencephalitis and nonspecific white matter lesions.
(B) T2/FLAIR hyperintensity in the brainstem sparing the red nuclei and substantia nigra commonly
is described in Wilson disease. (C) A leading edge
of restricted diffusion can be seen in large or new
lesions in progressive multifocal leukoencephalopathy. (D) Lesions involving the pulvinar and
dorsomedial thalamic nuclei are characteristic
of variant Creutzfeldt-Jakob disease. (E) Active
demyelination can present with an incomplete
rim of enhancement.
A 54-year-old man underwent an MRI of the spine.
Sagittal and axial postcontrast T1-weighted images
are shown in these images. What is the patient’s
likely diagnosis?
A. Leptomeningeal metastasis
B. Neurofibroma
C. Schwannoma
D. Meningioma
D. Meningioma
The demonstrated homogeneously enhancing
dural-based mass along the left ventral aspect of
the spinal canal at the level of C2 is most consistent
with a meningioma. It is more common in females
and, when in the spine, it most frequently occurs
in the thoracic region. It nearly always is completely intradural, but also may protrude through
the neural foramina, resulting in a “dumbbell”
configuration and thus may look similar to schwannomas. (A) Leptomeningeal metastases usually
develop along the pial surface of the cord and spinal nerves. The lesion in question is dural based.
(B) Neurofibromas may be indistinguishable from
schwannomas by imaging. They may show a socalled target sign with central hypointensity on
T2-weighted sequences due to a fibrocollagenous
core. (C) Schwannomas tend to enhance more
avidly and heterogeneously than meningiomas,
particularly due to the presence of cystic changes
and hemorrhages in larger lesions. They follow the
course of the involved nerve and do not show a
dural base. Although not entirely specific, they can
extrude through and expand the neural foramina,
resulting in a “dumbbell” configuration.
The lesion shown in this image (arrow) can result
from injury to what structure?
A. Central tegmental tract
B. Lateral lemniscus
C. Spinothalamic tract
D. Reticulospinal tract
A. Central tegmental tract
The axial FLAIR image accompanying the question demonstrates hyperintensity of the left inferior olivary nucleus in a patient with hypertrophic
olivary degeneration. This can be caused by a
lesion involving the triangle of Guillain-Mollaret,
a circuit connecting the dentate, red, and inferior
olivary nuclei. Efferent fibers from the dentate
nucleus ascend via the superior cerebellar peduncle
and decussate to the contralateral red nucleus, from
which fibers project inferiorly to the ipsilateral
inferior olivary nucleus through the central tegmental tract. Patients characteristically present
with palatal tremors. (B) The lateral lemniscus is
not part of the Guillain-Mollaret triangle. Bilateral
lesions are associated with hearing loss. (C) The
spinothalamic tract is a sensory pathway that
transmits pain and temperature sensation from the
spinal cord to the thalami. (D) The reticulospinal
tract transmits information from the reticular formation in the pons and medulla to the spinal cord.
It is not part of the Guillain-Mollaret triangle.
Axial T2 and postcontrast T1-weighted imaging of
the lumbar spine are shown in these images. To
what does the abnormality indicated by the arrow
correspond?
A. Disk protrusion
B. Epidural scar
C. Disk extrusion
D. Epidural abscess
E. Sequestered disk
B. Epidural scar
The axial T2 images accompanying the question show hypointense tissue projecting into the
right paracentral zone. This tissue shows diffuse
enhancement on the postcontrast image, and therefore is most consistent with scar. Note the evidence
of a prior right-sided laminotomy. (A, C, E) A disk
protrusion or extrusion or a sequestered disk
should not enhance. (D) There is no fluid collection
with peripheral enhancement to suggest an epidural abscess.
A 12-year-old girl is brought to the emergency
department in an obtunded state following an
episode of seizures. Based on these images, what
is the diagnosis?
A. Cortical venous thrombosis
B. Deep venous thrombosis
C. Mitochondrial encephalopathy
D. Hypoxic ischemic encephalopathy
E. Arterial infarction
B. Deep venous thrombosis
The sagittal noncontrast T1-weighted image
(right) accompanying the question shows increased
signal in the straight sinus, vein of Galen, and
internal cerebral veins due to thrombosis. Also note
the profound hypointensity of the thalami compared with the brain. The axial FLAIR image (left)
shows marked swelling of the basal ganglia and
thalami bilaterally due to venous infarction. Risk
factors for deep venous thrombosis include severe
dehydration and other hypercoagulable states such
as pregnancy, malignancy, and sepsis. Venous
infarction occurs in a nonarterial distribution and
may be complicated by hemorrhage. (A) Cortical
venous thrombosis may lead to lobar infarctions
sparing the deep gray structures. (C) Leigh disease
is a mitochondrial encephalopathy that may affect
the basal ganglia, periaqueductal gray, and cerebral peduncles. Changes in the putamina seem
to be a consistent feature. (D) Hypoxic ischemic
encephalopathy can occur following hypoxia, such
as in cardiorespiratory arrest, drowning, or various
forms of asphyxiation. In older children and adults,
the watershed zones initially may be affected after
mild insults, with more severe cases involving gray
matter structures, particularly the cerebral cortex,
basal ganglia, and hippocampi. Perinatal hypoxic
ischemic injury preferentially may affect the thalami, brainstem, and perirolandic cortex. There may
be white matter involvement in the setting of
global ischemia. (E) The imaging abnormalities in
this case do not follow an arterial distribution.
A woman underwent a head CT, shown in these
images. What is the likely diagnosis?
A. Infiltrative tumor
B. Acute infarct
C. Intracranial hemorrhage
D. Meningitis
C. Intracranial hemorrhage
There is a hemispheric subdural hematoma along
the right convexity that is isodense to the cortex.
This appearance can occur depending on when the
patient is imaged, as the density of blood decreases
over time and, at some point, will have the same
attenuation as cortex. The best way to recognize
such hematomas is to identify their associated mass
effect and the displacement of the darker white
matter, which can be seen medial to the hematoma. (A) An infiltrative tumor would involve the
cortex and white matter more diffusely. The demonstrated process is centered in the extra-axial space
and is subdural due to its overall crescentic shape
and the fact that it crosses sutures. (B) A noncontrast head CT in acute infarct may be normal, particularly in the first 6 hours after ictus. Early
findings include a hyperdense middle cerebral
artery territory, loss of gray matter–white matter
distinction in the basal ganglia or peripheral brain,
and loss of the insular ribbon. (D) The majority
of noncontrast head CT studies in meningitis are
normal, and patients sometimes may present with
hydrocephalus as an isolated finding. Areas of
edema due to cerebritis or intra- and extra-axial
abscesses may be seen in complicated meningitis.
Contrast studies, in particular MRI, may be able to
show the presence of leptomeningeal disease.
A boy with truncal ataxia and abnormal eye movements undergoes an MRI of the brain, which shows
continuation of the cerebellar hemispheres and
dentate nuclei and absence of the vermis. What is
the most likely diagnosis?
A. Pontine tegmental cap dysplasia
B. Rhombencephalosynapsis
C. Joubert syndrome
D. Dandy-Walker malformation
B. Rhombencephalosynapsis
Rhombencephalosynapsis is characterized by
an absent vermis and midline continuation of
the dentate nuclei, cerebellar hemispheres, and
superior cerebellar peduncles. It can be seen in
isolation or associated with other malformations
such as the VACTERL spectrum and Gomez-LopezHernandez syndrome. (A) Pontine tegmental cap
dysplasia is a rare hindbrain malformation that includes dysplasia of the cerebellar vermis, lateralized superior cerebellar peduncles, ectopic dorsal
transverse pontine fibers (tegmental cap), flattened ventral pons, absence of the inferior olives,
and absence or near absence of the middle cerebellar peduncles. (C) Joubert syndrome is characterized by vermian hypoplasia or aplasia and lack of
decussation of the superior cerebellar peduncles,
resulting in a “molar tooth” appearance. (D) A
Dandy-Walker malformation can result in cystic
enlargement of the posterior fossa in communication with the fourth ventricle. The vermis is
hypoplastic, but there is no continuation of the
cerebellar structures.
A 38-year-old man who sustained a gunshot wound
to the head underwent an emergent CT scan,
shown in this image. What finding portends the
worst prognosis?
A. Presence of both entry and exit wounds
B. Involvement of the inner and outer tables of
the calvaria
C. Bullet tract crossing the deep midline structures
D. Presence of metallic fragments along the bullet trajectory
E. Presence of an open comminuted fracture
C. Bullet tract crossing the deep midline structures
Various studies consistently have shown that
bullet tracts crossing the midline (i.e., bihemispheric involvement) are associated with increased
mortality and worse functional outcomes in those
patients who survive, although there are recent
data suggesting that individuals with isolated
bifrontal involvement may have a relatively better
prognosis. Other significant negative prognostic
factors include brainstem involvement, posterior
fossa injuries, transventricular injuries, low Glasgow
Coma Scale score, nonreactive pupils, and older age.
(A) The presence of both entry and exit wounds
is not a significant predictor of worse outcomes.
(B) Involvement of the inner and outer tables of
the calvaria is not a significant predictor of worse
outcomes. (D) The presence of bullet fragments
is not a significant predictor of worse outcomes.
(E) The presence of an open comminuted fracture
is not a significant predictor of worse outcomes.
The axial CT scan in this image shows the level of
termination of bilateral cerebral deep brain stimulation leads. What is the anatomic location of these
leads?
A. Globus pallidus interna
B. Subthalamic nuclei
C. Ventral intermediate nuclei
D. Red nuclei
A. Globus pallidus interna
The image accompanying the question shows
bilateral deep brain stimulator leads terminating
in the globus pallidus interna (GPi) that are used
to treat motor fluctuations in advanced medication
resistant Parkinson disease or levodopa induced
dyskinesia. (B) The subthalamic nuclei are an alternative target for deep brain stimulation in Parkinson disease. They are located at a lower level
superior to the substantia nigra, lateral to the red
nuclei, and medial to the internal capsule. (C) The
ventral intermediate nuclei are located in the ventral thalami just lateral to the red nuclei. They are a
common target for the treatment of essential tremors. (D) The red nuclei are paired, round T2 hypointense structures located in the rostral midbrain.
A man is reported to have a “string of pearls”
appearance on his angiogram. What is the likely
diagnosis?
A. Severe carotid artery stenosis
B. Dural arteriovenous fistula
C. Fibromuscular dysplasia
D. Arteriovenous malformation
E. Carotid artery dissection
C. Fibromuscular dysplasia
The medial type of fibromuscular dysplasia is by
far the most common and can show a “string of
pearls” appearance on angiography. The same descriptor also is sometimes used for cerebral vasculitis, which can show areas of alternating segmental
vascular stenoses and dilatation. (A) A “string sign”
sometimes is described with severe carotid artery
stenosis due to a very thin column of contrast flowing through the narrowed segment. (B) The “buzz”
term for a dural arteriovenous fistula is “retrograde cortical venous drainage” or “cortical venous
reflux,” which is associated with more aggressive
behavior and an indication for emergent treatment. (D) Arteriovenous malformations display early
draining veins and the presence of a nidus sometimes with venous or arterial aneurysms. (E) Carotid artery dissections can show a “string sign” in the narrowed segment; however, dissection of the internal carotid artery typically is associated with a “flame shape” configuration when it occurs just above the level of the carotid bulb.
A sagittal T2-weighted image of a patient with a
tethered cord is shown in this image. What is a
characteristic of the pathology represented here?
A. Reduced risk following folic acid supplementation
B. Associated with Chiari 2 malformations
C. Secondary to premature disjunction of the
neural ectoderm
D. Most cases are familial
C. Secondary to premature disjunction of the
neural ectoderm
Lipomyelomeningoceles, lipomyeloceles, and
lipomas are secondary to premature disjunction of
the neural ectoderm from the cutaneous ectoderm.
(A) Studies have not found a decreased incidence of
lipomyelomeningoceles following folic acid supplementation, suggesting that the pathogenesis
is different from that of other neural tube defects.
(B) Open myelomeningoceles are the sine qua non
of Chiari 2 malformations. Closed defects covered
by skin such as this one do not result in Chiari 2
malformations. (D) The vast majority of lipomyelomeningoceles are sporadic. A few familial cases
have been reported, but they are exceedingly rare.
What is the origin of the lesion on the contrastenhanced T1 image shown here?
A. Facial nerve
B. Vestibular nerve
C. Aberrant carotid artery
D. Inferior petrosal sinus
A. Facial nerve
There is an avidly enhancing mass involving the
right geniculate ganglion as well as the labyrinthine and canalicular segments of the right facial
nerve compatible with a schwannoma. The vast
majority of intracranial schwannomas arise from
the vestibular nerve followed by the trigeminal and
facial nerves. (B) A vestibular schwannoma would
not involve the facial nerve canal or geniculate
ganglion. (C) An aberrant internal carotid artery is
seen more inferiorly as a more lateral extension
of the internal carotid artery beyond the cochlear
promontory. It may appear as a pulsating “mass”
on clinical exam. (D) The inferior petrosal sinus
is located along the inferior aspect of the petrous
bone. It drains blood from the cavernous sinus into the jugular vein
A falcotentorial arteriovenous malformation is
noted to have its primary vascular supply from an
enlarged tentorial artery. What is the usual origin
of this vessel?
A. Meningohypophyseal trunk
B. Inferolateral trunk
C. Neuromeningeal trunk
D. Posterior cerebral artery
A. Meningohypophyseal trunk
The tentorial artery (of Bernasconi and Cassinari) is the most constant branch of the meningohypophyseal trunk, which in turn arises from the
cavernous internal carotid artery. It is an important structure due to its vascular supply to lesions
in the region of the tentorium cerebelli, such as
vascular malformations and meningiomas. (B) The
inferolateral trunk arises along the lateral aspect of the cavernous internal carotid artery and projects
inferiorly. It usually has three or four branches and
multiple anastomoses with branches of the external carotid artery. (C) The neuromeningeal trunk
is a branch of the ascending pharyngeal artery
and consists of jugular and hypoglossal divisions.
(D) The posterior cerebral arteries most commonly arise as bifurcations of the basilar artery
Sagittal CT and MRI STIR sequences of the cervical
spine are shown in these images. What is the
injury type demonstrated?
A. Hangman fracture
B. Clay-shoveler fracture
C. Jefferson fracture
D. Flexion-distraction injury
E. Locked facets
D. Flexion-distraction injury
The fracture demonstrated is a classic case of a
severe flexion-distraction injury with a teardroptype fracture of the anteroinferior C4 vertebra as
well as distraction of the C4-C5 facets and posterior elements as can be seen on the CT images. The
STIR image shows extensive edema in the posterior
soft tissues including the interspinous ligaments,
which are disrupted. These are highly unstable
fractures associated with acute anterior cervical
cord syndrome. (A) A hangman (or more correctly
“hangee”) fracture involves both pedicles or both
partes interarticulares of C2 and is secondary to
hyperextension and distraction. (B) A clay-shoveler
fracture is a stable fracture involving a lower cervical vertebra (usually C7) and sometimes the
upper thoracic spine. The fracture is a type of hyperflexion avulsion injury. (C) A Jefferson fracture
is a burst fracture of the C1 vertebra. It is considered unstable if the combined offset of the lateral
C1 masses measures more than 7 mm or if the
atlantodental distance measures more than 3 mm.
(E) Locked facets may be a result of a flexiondistraction injury and can result in listhesis. The
C4-C5 facets in this case are mildly distracted.
What MRI features would favor a metastatic (pathological) compression fracture over a benign osteoporotic fracture?
A. Horizontal low signal intensity bands
B. Convex posterior vertebral margins
C. Areas of spared vertebral marrow
D. Retropulsion of a bone fragment
E. Enhancement of the involved vertebra
B. Convex posterior vertebral margins
Convex posterior vertebral margins are suggestive of metastatic disease. Other features that would
support this etiology are epidural or paravertebral
masses or the presence of vertebral metastases at
other levels. Metastases tend to involve the pedicle
and posterior elements more commonly, but these
sites also can show edema related to benign fractures. (A) Horizontal low signal intensity bands can be seen on both T1- and T2-weighted images and are more common in benign osteoporotic fractures.
(C) Areas of spared vertebral marrow (best seen as
high signal on noncontrast T1 images) are seen
more commonly in benign compression fractures.
(D) Retropulsion of a bone fragment is more suggestive of a benign compression fracture. (E) Both benign and pathological compression fractures can be accompanied by significant enhancement.
A man with a history of depression presents with
rapid and involuntary movements involving his
face and limbs. The clinical exam is notable for
hypotonia, hyperreflexia, and mild bradykinesia. A
noncontrast CT of the head is shown in this image.
What mutation is the likely cause of the patient’s
presentation?
A. Trinucleotide repeat expansion
B. Point mutation
C. Frameshift mutation
D. Deletion
A. Trinucleotide repeat expansion
The CT shows atrophy of the heads of the caudate nuclei bilaterally, resulting in abnormal ballooning of the frontal horns of the lateral ventricles,
which is consistent with Huntington disease, given
the patient’s clinical presentation. Huntington
disease results from trinucleotide (CAG) repeat expansion in the huntingtin gene, which is located on
chromosome 4. (B) Point mutations are modifications of a single nucleotide base and include substitutions, deletions, and insertions. (C) Frameshift
mutations result from insertions, deletions, or
duplications that alter the normal trinucleotide
reading frame. (D) Deletions indicate of loss of
genetic material and can involve individual bases or an entire gene
Gradient echo MRI sequences are particularly useful for the detection or evaluation of what process
or pathology?
A. Myelin injury
B. Purulence
C. Acute ischemia
D. Glucose metabolism
E. Blood products
E. Blood products
Gradient echo (GRE) sequences generate images
that particularly are susceptible to magnetic field
inhomogeneities such as those produced in the
presence of paramagnetic blood degradation products (e.g., hemosiderin and ferritin). The presence
of these products manifests as signal loss and can
be used to detect hemorrhage. Dynamic susceptibility contrast MRI and functional MRI are different techniques based on GRE sequences. (A) Myelin
injury is difficult to demonstrate on conventional
MRI unless there is clear disruption of the white
matter. Certain metrics based on advanced MRI
techniques such as radial diffusivity (RD) in diffusion tensor imaging (DTI) may serve as indicators
of primary myelin injury. A different technique
called magnetization transfer can provide a measure of the contribution of protons that are bound
to macromolecules (e.g., myelin) and has been employed in the evaluation of demyelinating disorders. (B) Diffusion-weighted imaging (DWI) with
apparent diffusion coefficient (ADC) maps is very
sensitive for the detection of purulence within
abscesses, cavities, or the ventricular system.
(C) Acute ischemia is best demonstrated with DWI
and ADC maps. (D) Glucose metabolism can be
assessed with positron emission tomography (PET)
by using fluorodeoxyglucose (FDG).
A man without a history of trauma is brought to
the emergency department with nausea, vomiting,
and ataxia. Axial T2-weighted images of the neck
and posterior fossa are shown here. What is a characteristic of the lesion in the neck?
A. It may be related to connective tissue disorders.
B. The majority occur in patients older than 60
years of age.
C. Intradural lesions are more common.
D. Rupture is more common in extradural than
intradural lesions
A. It may be related to connective tissue disorders.
The image on the left shows somewhat crescentic and nearly circumferential mural thickening of
the left vertebral artery in a patient with a spontaneous dissection (compare with the normal dark
vascular flow void on the right). The image on the
right shows cerebellar infarcts. Minor or sometimes unrecalled trauma may account for some
cases of “spontaneous” dissection, and there is an
increased incidence in patients with connective
tissue disorders. (B) Vertebral artery dissection
occurs in young and middle age adults with a mean
age of presentation of 40 years of age. (C) Extradural vertebral artery dissections are more common than intradural ones. (D) Rupture is more
common in intradural vertebral dissections because
of a lack of external elastic lamina, fewer elastic
fibers in the media, and a thinner adventitial layer.
A 56-year-old patient involved in a motor vehicle
collision underwent a cervical spine CT scan, shown
in these images. What is true of the osseous abnormality demonstrated?
A. It is consistent with an acute type 2 odontoid
fracture.
B. It results from failure of fusion of the ossiculum terminale.
C. It is associated with Morquio syndrome and
multiple epiphyseal dysplasia.
D. Orthotopic lesions are more likely to be unstable than dystopic lesions.
C. It is associated with Morquio syndrome and
multiple epiphyseal dysplasia.
An os odontoideum is a rare anomaly of the
cervical spine characterized by a deformity of the
odontoid process that appears as a smooth and
well-corticated ossicle separated from the body
of C2. There may be hypertrophy of the anterior
C1 arch, as seen in this case. It may be orthotopic
where the ossicle is in a relatively anatomic location with a gap between it and the body of C2, or
dystopic where the ossicle is located in any other
position. An increased frequency of os odontoideum has been reported in patients with multiple
epiphyseal dysplasia and in Morquio, Down, and
Klippel-Feil syndromes. (A) The ossicle above the
body of C2 is round and features smooth, wellcorticated margins; therefore, the abnormality
shown is not an acute fracture. (B) Failure of fusion
of the ossiculum terminale usually results in a much
smaller ossicle above an overall normal-appearing
odontoid process (except for a small notch at the
tip). The ossiculum terminale normally fuses with
the body of the dens around age 12. When persistent, it usually is asymptomatic and rarely associated with instability. (D) An os odontoideum usually
is asymptomatic and, in many instances, is discovered incidentally; however, it may be associated
with instability due to hypermobility of C1 over
C2. This immobility may lead to spinal cord or, less
frequently, vertebral artery compression. Dystopic
lesions are more likely to be unstable than orthotopic lesions.
A 52-year-old obese woman with a history of headaches presents to clinic. An MRI of the brain was
obtained, and is shown in these images. What additional radiographic finding may be seen?
A. Dural venous sinus stenosis
B. Venous sinus engorgement
C. Brainstem sagging
D. Decreased mammillopontine distance
A. Dural venous sinus stenosis
The axial T2 image shows distention of the optic
nerve sheaths and prominence of the optic cups bilaterally due to an increased intracranial pressure and papilledema in a patient with idiopathic
intracranial hypertension (pseudotumor cerebri).
The sagittal CISS image on the right shows an
expanded and partially empty sella turcica. Additional findings in these patients include dural
venous sinus stenosis, enlargement of the Meckel
caves, and cerebrospinal fluid arachnoid pits in
the sphenoid bones. (B) Venous sinus engorgement is seen in intracranial hypotension, not hypertension, due to decreased cerebrospinal fluid volumes (and in keeping with the Monro-Kellie doctrine). (C) Brainstem sagging is a feature of intracranial hypotension. (D) Decreased mammillopontine distance is a sign of brainstem sagging in
intracranial hypotension.
A woman who was involved in a motor vehicle collision underwent a CT of the cervical spine, which
shows a fracture involving an occipital condyle.
What structure is likely to be affected?
A. Cranial nerve IX
B. Cranial nerve X
C. Cranial nerve XI
D. Cranial nerve XII
D. Cranial nerve XII
The hypoglossal canals course along the medial
and superior aspects of each occipital condyle between the basiocciput and the jugular process.
They transmit cranial nerve XII, which is at high
risk of injury in the presence of condylar fractures.
(A) Cranial nerve IX courses through the pars nervosa of the jugular foramen along with the inferior
petrosal sinus. (B, C) Cranial nerves X and XI course
through the pars vascularis of the jugular foramen
along with the jugular bulb.
The lesion shown in this image is associated with a
chromosome 13q deletion. What is a characteristic
of this lesion?
A. Has a tendency for leptomeningeal spread
B. Anterior eye segment enhancement indicates
disease infiltration
C. Likely has high apparent diffusion coefficient
values
D. Likely has high signal intensity on susceptibility-weighted imaging
A. Has a tendency for leptomeningeal spread
The axial T2 image shows a T2 hypointense mass
involving the left posterior globe and left optic
nerve extending posteriorly to the level of the chiasm. There are foci with even lower signal intensities within the globe consistent with hemorrhage.
Although an optic glioma may cause enlargement
of the optic nerve, involvement of the eye and
deletion of chromosome 13q (which contains the
RB1 gene locus) indicates a diagnosis of retinoblastoma. This is a highly malignant tumor that has a
tendency for leptomeningeal spread. (B) Anterior
eye segment enhancement generally represents
reactive angiogenesis. (C) Retinoblastomas, as with
other primitive neuroectodermal tumors, commonly
have profoundly low apparent diffusion coefficient
values due to high cellularity and restricted diffusion. (D) The lesion shown likely has areas of low
signal intensity on susceptibility-weighted imaging
due to calcifications and hemorrhage.
What is true regarding the lesion shown in this CT
image?
A. Usually heals well with traction and immobilization
B. Usually considered stable
C. Constitutes the most common type of fracture
at this site
D. Fracture occurs above the transverse band of
the cruciform ligament
E. Most likely fracture type to progress to
nonunion
A. Usually heals well with traction and immobilization
This is a type 3 fracture of the odontoid process
extending from its base into the body of C2. These
fractures are unstable, as they allow the odontoid
process to move with the occiput as a unit; however, they have the best prognosis of all odontoid
fracture types. They usually heal well with traction
followed by bracing. (B) Although type 3 odontoid fractures have the best prognosis of all odontoid fracture types, they are considered unstable.
(C) Type 2 odontoid process fractures are the most
common. They occur across the base of the odontoid process at its junction with the body of C2.
(D) Type 3 odontoid fractures occur inferior to the
level of the transverse band of the cruciform ligament. (E) Type 2 odontoid process fractures are the
most likely to progress to nonunion.
A patient with left-sided cranial neuropathies
underwent a brain MRI study, shown in these
images. What is a likely complication of this
lesion?
A. Malignant transformation
B. Carotid-cavernous fistula
C. Subarachnoid hemorrhage
D. Posterior circulation infarcts
B. Carotid-cavernous fistula
The axial precontrast T1 image on the left shows
a round, mixed signal intensity lesion in the left
cavernous sinus. Note a subtle horizontal band at
the level of the lesion propagating in the phaseencoding direction compatible with pulsation
artifact. The T2 image on the right shows areas of
profound hypointensity. Findings are indicative of
an aneurysm, which probably is partially thrombosed. Compared with anterior and posterior circulation aneurysms, cavernous carotid aneurysms
have the lowest rate of rupture (posterior circulation aneurysms have the highest). Rupture of
cavernous carotid aneurysms may result in direct
carotid-cavernous fistulae. (A) Malignant transformation may be seen in certain neoplasms. The
lesion in this case is an aneurysm. (C) Subarachnoid hemorrhage can occur from rupture of an
intradural aneurysm. (D) Posterior circulation
infarcts can be the result of embolic phenomena
from aneurysms located more proximally in the
posterior circulation.
What is a characteristic of the entity depicted on
the digital subtraction angiographic images shown
here?
A. Cortical venous reflux denotes increased bleeding risk.
B. Multiple lesions may be seen in KlippelTrenaunay-Weber syndrome.
C. Deep venous drainage connotes increased surgical risk.
D. Most have a primarily dural vascular supply
C. Deep venous drainage connotes increased surgical risk.
With arteriovenous malformations, deep venous
drainage, involvement of eloquent cortex, and larger
nidal sizes connote an increased surgical risk and
are part of the Spetzler-Martin grading system.
(A) Cortical venous reflux is associated with an
increased bleeding risk in patients with dural arteriovenous fistulas. The patient in this case has a
pial arteriovenous malformation with a large nidus
supplied from branches of the anterior and middle
cerebral arteries. (B) Most arteriovenous malformations (98%) are solitary, but they rarely may
be multiple particularly in patients with certain
syndromes such as Osler-Weber-Rendu (who also
can have microaneurysms), Wyburn-Mason, and
craniofacial arteriovenous metameric syndromes
(CAMS). Klippel-Trenaunay-Weber syndrome is
characterized by capillary, venous, and lymphatic
malformations as well as soft tissue and osseous
hypertrophy. (D) Most arteriovenous malformations
(75%) have a primary pial blood supply from the internal carotid artery; 15% have dual supply from the external carotid artery; 10% receive blood from both the internal and external carotid arteries.
What is a characteristic of the injury shown on this
CT scan?
A. Combined offset of the lateral C1 masses relative to C2 greater than 6 mm, which suggests
disruption of the alar ligaments
B. Frequently associated with diving head first into shallow water
C. High frequency of neurologic injury
D. Most commonly occurs in infants and young
children
E. Usually warrants emergent surgical fixation
B. Frequently associated with diving head first into shallow water
This is a burst fracture of C1 (also known as Jefferson fracture) that develops due to axial loading
transmitted through the occipital condyles into
the C1 vertebra and is seen frequently after diving
head first into shallow water. Jefferson fractures
tend to be stable, and neurologic deficits are infrequent unless there is disruption of the transverse
ligament, fractures at other levels, or injury to the
vertebral arteries. Disruption of the transverse ligament is suspected if there is widening of the
atlantodens interval (normally less than 3 mm in
adults and 5 mm in children) or if the combined
offset of the lateral masses of C1 relative to the lateral C2 pillars measures greater than 6 mm. If the
transverse ligament is disrupted, stability depends
on the alar ligaments connecting the dens with the
medial aspect of the occipital condyles. (A) Combined offset of the lateral C1 masses relative to
C2 greater than 6 mm suggests disruption of the transverse ligament and raises concern about an unstable fracture. (C) Jefferson fractures have a low frequency of neurologic injury unless there is disruption of the transverse ligament, fractures at other levels, or injury to the vertebral arteries.
(D) Jefferson fractures are extremely rare in infants
and young children, probably due to the plasticity
of their bones and the presence of soft synchondroses. (E) These fractures usually are managed conservatively unless they are unstable.
The abnormality in the left frontal lobe on the CT
of the head shown in this image is consistent with
what process?
A. Acute infarction
B. Cortical contusion
C. Remote injury
D. Neoplasm
E. Abscess
C. Remote injury
The area of hypoattenuation in the left frontal
lobe is consistent with encephalomalacia. It demonstrates very low attenuation and is associated
with volume loss, as can be seen by ex vacuo expansion of the frontal horn of the left lateral ventricle. This likely is secondary to remote trauma,
infarction, or other injury. This patient also has an
acute, right frontal, extra-axial hematoma. (A, B)
Acute or subacute infarction or cortical contusion
would not be associated with volume loss, and the
lesion margins would likely be less well defined.
(D) A neoplasm would show some degree of mass
effect and may have surrounding edema. (E) There
is no fluid collection or evidence of vasogenic
edema to suggest an abscess.
What entity is compatible with the abnormality
indicated by the arrow on the MRI shown in this
image?
A. Ependymoma
B. Neuromyelitis optica
C. Subacute combined degeneration
D. Infectious myelitis
C. Subacute combined degeneration
Subacute combined degeneration is caused by
vitamin B12 deficiency, and leads to demyelination
and vacuolization of the dorsal columns of the spinal cord with or without involvement of the lateral
columns. It most commonly occurs in the upper
thoracic and cervical cord. MRI shows T2 signal
hyperintensity with rare contrast enhancement.
Subacute combined degeneration can be exacerbated or caused by nitrous oxide toxicity, which
temporarily interrupts metabolism of B12. (A) There
is no evidence of a space-occupying lesion to suggest a neoplasm. Ependymomas tend to occur centrally within the spinal cord, and cysts, hemorrhage,
and calcifications are common findings. (B) Neuromyelitis optica results in greater cord expansion
than demonstrated in the image and usually is
more central and not confined to the dorsal columns. (D) Infectious myelitis is rare, and imaging
may show edema that is not confined to the dorsal columns.
A patient with extensive T2 hyperintensity and
enhancement of the skull base shows a “black turbinate” sign on postcontrast T1 MRI sequences.
What is the likely etiology of these findings?
A. Bacterial infection
B. Invasive fungal infection
C. Nasopharyngeal carcinoma
D. Osseous infarction
B. Invasive fungal infection
The “black turbinate” sign refers to nonenhancing nasal mucosa due to tissue infarction, and has
been described in angioinvasive fungal infections. The skull base findings in this case are secondary
to fungal osteomyelitis. (A) Bacterial infection can
lead to osteomyelitis of the skull base but does not
result in a “black turbinate” sign. (C) Nasopharyngeal carcinoma can invade the skull base but does
not produce a “black turbinate” sign. (D) Osseous
infarctions can present with mixed signal intensities on T2-weighted MRI sequences due to areas of
edema and sclerosis and irregular enhancement.
Skull base infarcts are rare but occur more commonly in patients with sickle cell disease, in which case they can be complicated by osteomyelitis.
What explains the development of the abnormality demonstrated on the brain MRI shown in these
images?
A. Failure of closure of the rostral neuropore
B. Failure of diverticulation
C. Nondisjunction of the neural ectoderm
D. Premature disjunction of the neural ectoderm
B. Failure of diverticulation
These images show nonseparation of the frontal
lobes and an azygous anterior cerebral artery in a
patient with lobar holoprosencephaly. These occur
secondary to the failure of diverticulation and
cleavage of the prosencephalon during embryonic
life. Holoprosencephaly represents a spectrum of
abnormalities, with the alobar form being the most
severe and commonly presenting with a monoventricle, thalamic fusion, and facial anomalies.
(A) Failure of closure of the rostral neuropore leads
to anencephaly. (C) Nondisjunction of the neural tube can result in open spinal dysraphisms. (D) Premature disjunction of the neural tube can lead to lipomyeloceles, lipomeningoceles, and lipomyelomeningocele
What tumor type would be expected to show the
lowest apparent diffusion coefficient (ADC) values
on an MRI of the brain?
A. Atypical teratoid rhabdoid tumor
B. Juvenile pilocytic astrocytoma
C. Ependymoma
D. Dysembryoplastic neuroepithelial tumor
E. Diffuse astrocytoma
A. Atypical teratoid rhabdoid tumor
Atypical teratoid rhabdoid tumors and other
embryonal tumors as well as medulloblastomas
are highly malignant and typically show the lowest
apparent diffusion coefficient (ADC) values due to
their high cellularity. (B) Juvenile pilocytic astrocytomas are WHO grade I tumors that do not show
significant restricted diffusion. (C) Ependymomas
may show variable degrees of restricted diffusion,
but their ADC values typically are higher than those
of embryonal tumors. (D) Dysembryoplastic neuroepithelial tumors are benign glioneuronal neoplasms (WHO grade I) that do not show restricted
diffusion; therefore, their ADC values are high.
(E) Diffuse astrocytomas are WHO grade II lesions
that do not show significant restricted diffusion.
What is a characteristic of acute demyelinating
encephalomyelitis?
A. Most cases occur following vaccination.
B. Deep gray nuclei usually are spared.
C. Most lesions show contrast enhancement.
D. It is typically a monophasic process.
D. It is typically a monophasic process.
Acute demyelinating encephalomyelitis (ADEM)
usually is a monophasic process, but some patients
are at an increased risk of recurrence (defined as a
new event after 3 months) or the development of
multiple sclerosis. (A) Approximately 75% of cases
of ADEM occur after viral and less commonly bacterial (e.g., Campylobacter) infections. (B) The deep
gray nuclei (particularly the thalami) are involved
in 60% of patients with ADEM. These structures are
involved less commonly in multiple sclerosis. (C)
Only 25% of ADEM lesions show contrast enhancement. There usually is no mass effect except for
cases of tumefactive demyelination, which also may be accompanied by a leading edge of restricted diffusion.
A 5-year-old boy with visual and hearing deficits
and loss of developmental milestones presents for
evaluation. MRI of the brain shows occipitoparietal periventricular demyelination with a leading
edge of enhancement. What is the patient’s likely
diagnosis?
A. Adrenoleukodystrophy
B. Canavan disease
C. Alexander disease
D. Krabbe disease
E. Pelizaeus-Merzbacher disease
A. Adrenoleukodystrophy
The presentation and imaging findings are consistent with X-linked adrenoleukodystrophy. This
disease typically occurs in 4- to 6-year-old boys and
is caused by mutations in the ABCD1 gene, leading
to accumulation of very long chain fatty acids.
Death or progression to a vegetative state usually
occurs 2 years after symptom onset. (B) Canavan
disease is caused by mutations in the ASPA gene on
chromosome 17. It results in macrocephaly, with
imaging showing sequelae of white matter demyelination and signal abnormalities in the globi pallidi and thalami. (C) Alexander disease is caused by
mutations in the GFAP gene on chromosome 17. It
is characterized by demyelination that progresses
from anterior to posterior. Patients have astrocytic Rosenthal fibers and macrocephaly. (D Krabbe
disease results from mutations in the GALC gene
on chromosome 14 and leads to the deficiency of
galactocerebroside β-galactosidase. The condition
most commonly is diagnosed between the third
and sixth months of life. Imaging shows abnormalities in the white matter, basal ganglia, cerebellum,
and corticospinal tracts. (E) Pelizaeus-Merzbacher
disease is an X-linked recessive disorder characterized by oligodendrocyte dysfunction leading to
hypomyelination. Patients usually are males and
present with nystagmus, seizures, hypotonia, and
ataxia. On MRI, there is diffuse T2 signal hyperintensity in the white matter due to hypomyelination (myelinated white matter is dark on T2 and bright on T1 sequences).
The spinal cord lesion in the T2-weighted MRI
shown in this image is consistent with what disease process?
A. Poliomyelitis
B. Amyotrophic lateral sclerosis
C. Guillain-Barré syndrome
D. Subacute combined degeneration
D. Subacute combined degeneration
Subacute combined degeneration can be caused
by vitamin B12, copper, or vitamin E deficiencies.
It results in an abnormal signal within the dorsal
columns of the spinal cord in an inverse V-shaped
configuration with variable involvement of the lateral columns. Abuse of nitrous oxide can produce
the same syndrome through inactivation of vitamin B12. Neurosyphilis (tabes dorsalis) is also
known to involve the dorsal columns. (A) Poliomyelitis may show an abnormal signal and enhancement of anterior horn cells as well as ventral roots
of the cauda equina if accompanied by radiculitis.
(B) Amyotrophic lateral sclerosis preferentially
involves the anterior and lateral columns of the
spinal cord, which may show increased T2 signal.
(C) Guillain-Barré syndrome presents with preferential pial enhancement of the conus medullaris
and dorsal roots of the cauda equina. It should
not be associated with intramedullary signal
abnormalities.
A brain MRI shows a small lesion in the pons that
is slightly hyperintense on T2-weighted imaging
and shows “brush-like” enhancement and signal
dropout on gradient echo sequences. There is no
surrounding edema. This lesion is consistent with
what disease process?
A. Demyelinating plaque
B. Metastasis
C. Cavernous malformation
D. Capillary telangiectasia
D. Capillary telangiectasia
The imaging description of this lesion and its
location are characteristic of capillary telangiectasias. Low signal on gradient echo (GRE) sequences
is thought to be related to slow flow and increased
deoxyhemoglobin content. These lesions sometimes can have areas of gliosis, and they nearly
always are asymptomatic and found incidentally.
(A) A demyelinating plaque does not show the
constellation of features described, which are characteristic of capillary telangiectasias. (B) Capillary
telangiectasias should be differentiated from
metastases. The latter usually have surrounding
edema (except for small lesions) and do not show
signal loss on GRE sequences unless they are hemorrhagic, melanotic, or calcified. (C) Cavernous malformations may show various signal intensities
depending on the stage of their blood products.
They typically are described as having a “popcorn” or “berry” appearance with a surrounding rim of hemosiderin.
Where is a type C carotid-cavernous fistula located
according to the Barrow classification?
A. Between meningeal branches of the external carotid artery and cavernous sinus
B. Between meningeal branches of the internal carotid artery and cavernous sinus
C. Directly between the cavernous internal carotid artery and cavernous sinus
D. Between meningeal branches of both the exter- nal and internal carotid arteries and cavernous sinus
A. Between meningeal branches of the external carotid artery and cavernous sinus
Carotid cavernous fistulas can be direct or indirect. Direct fistulas (Barrow type A) develop
between the cavernous internal carotid artery and
cavernous sinus and are therefore high flow. They
are more frequent in younger males and occur secondary to trauma or rupture of a cavernous internal carotid aneurysm. Types B, C, and D occur
between meningeal branches of the internal or
external carotid arteries, are slow flow, and are
most frequently seen in women older than 50 years
of age. A type C carotid cavernous fistula occurs
between meningeal branches of the external carotid
artery and cavernous sinus. (B) A Barrow type B
fistula occurs between meningeal branches of the
internal carotid artery and cavernous sinus. (C) A Barrow type A fistula occurs directly between the cavernous internal carotid artery and cavernous sinus. (D) A Barrow type D fistula occurs between meningeal branches of both the external and internal carotid arteries and cavernous sinus.
What is a characteristic of the lesion depicted in
the MRI shown in these images?
A. Lined by arachnoid cells resulting in accumulation of cerebrospinal fluid
B. Most are intraventricular
C. Show complete signal suppression on MR
FLAIR sequences
D. Usually show areas of patchy enhancement
E. Associated with a risk of malignant transformation
E. Associated with a risk of malignant transformation
T2, FLAIR, and diffusion-weighted imaging (DWI)
MRI sequences are presented showing an epidermoid cyst. These lesions are characterized by very
bright DWI signal (presumably a combination of
restricted diffusion and T2 shine-through effect),
lack of contrast enhancement, and nonsuppression
on FLAIR. Epidermoid cysts arise from ectodermal
inclusions during neural tube closure and rarely
can degenerate into squamous cell carcinomas.
(A) Arachnoid cysts and not epidermoid cysts are
lined by arachnoid cells, resulting in accumulation
of cerebrospinal fluid. (B) Up to 50% of epidermoid
cysts are located in the cerebellopontine angle,
although they uncommonly may arise within the
ventricles. (C) Although signal in epidermoid cysts
may resemble cerebrospinal fluid on T2, they do
not suppress on FLAIR as demonstrated in the
second image. The signal of epidermoid cysts on
FLAIR sometimes has been described as “dirty”
appearing and used to be the main imaging discriminator for diagnosis before DWI was available.
This is in contradistinction to arachnoid cysts,
which do suppress on FLAIR and follow cerebrospinal fluid signal on all sequences. (D) Although
there sometimes can be minimal marginal contrast enhancement, epidermoid cysts should not
show patchy or any significant enhancement, which
would be suspicious for malignant transformation.
A 12-year-old boy presented to the emergency
department with progressive headaches, nausea,
and vomiting. Axial T2 and contrast-enhanced T1-
weighted imaging and apparent diffusion coefficient maps are shown in these images. What is the
patient’s likely diagnosis?
A. Medulloblastoma
B. Ependymoma
C. Pilocytic astrocytoma
D. Hemangioblastoma
E. Metastasis
C. Pilocytic astrocytoma
A well-circumscribed cystic mass with an avidly
enhancing mural nodule in the cerebellum most likely is a juvenile pilocytic astrocytoma in a patient of this age, which has a peak incidence between 5 and 15 years of age. Despite its classification as WHO grade I, a pilocytic astrocytoma shows avid
contrast enhancement of the solid components and paradoxically may exhibit aggressive-appearing imaging characteristics, with a metabolite pattern mimicking a high-grade lesion on MR spectroscopy (increased choline, decreased N-acetylaspartate, and increased lactate) and increased metabolism
on 18F-fluorodeoxyglucose PET studies. A pilocytic astrocytoma most commonly occurs in the cerebellum followed by the optic nerve and chiasm (particularly when syndromic in neurofibromatosis type 1) and around the third ventricle and hypothalamus. It is a slow-growing tumor and most commonly presents with manifestations of increased intracranial pressure (such as in this case where there is hydrocephalus) and cerebellar signs.
(A) Medulloblastomas usually occur in patients
younger than 5 years of age but may be seen in
older individuals. They have low apparent diffusion coefficient (ADC) values, and therefore appear dark on the ADC maps (in contrast to this case, where the solid component of the tumor is bright). Medulloblastomas arise from the superior medullary velum and fill the fourth ventricle rather than displace it. (B) Ependymomas arise from the floor of the fourth ventricle and fill it rather than displace it, in contrast to the tumor of this case. Ependymomas characteristically extrude through
the ventricular outflow tracts into the foramen
magnum and posterior fossa with a “plastic” or
“toothpaste” appearance. They also tend to be seen in younger patients with a peak incidence between 1 and 5 years of age, although there is a smaller peak in young adults later in life. Cystic changes would be an unusual finding in ependymomas. (D) Hemangioblastomas more commonly present in adults and are rare in children unless they are syndromic (peak at 40 to 60 years of age). They also can present as a cystic mass with an enhancing mural nodule that typically abuts the pial surface, and they often have prominent flow voids due to hypervascularity. They may be associated with von Hippel–Lindau syndrome, but most are sporadic. (E) Some metastases may present with cystic changes, particularly from squamous cell primaries such as in head and neck, lung, and cervical cancers. Metastases, in general, are the most common cerebellar tumor in adults. They would be unexpected in a patient of this age
A 74-year-old woman presents to the emergency
room complaining of vertigo and nausea increasing in severity over the past several hours. CT and
MRI studies were performed, and representative
images are shown here. This lesion likely represents a(n):
A. Primary neoplasm
B. Acute infarction
C. Metastatic disease
D. Arachnoid cyst
E. Epidermoid cyst
B. Acute infarction
Noncontrast CT and diffusion-weighted images
are presented, and the distribution of the abnormality and presence of restricted diffusion are most
compatible with an acute posterior inferior cerebellar artery (PICA) territory infarct. PICA infarcts
may be associated with significant mass effect in
the posterior fossa and herniation with rapid
deterioration and death. Although they can present
with specific neurologic syndromes (such as lateral or medial medullary syndromes), symptoms
can be vague and nonspecific, and sometimes may
be confused with labyrinthitis or gastroenteritis.
(A) Hemorrhagic or high-grade primary neoplasms
may result in areas of restricted diffusion due to
blood products or high cellularity, respectively;
however, they would be expected to be more
heterogeneous and would not preserve the cerebellar architecture as in this case (note that some
of the folia still can be defined on the diffusion
image). Additionally, the vascular distribution of the
abnormality and the evolution of symptoms over
a few hours are compatible with a PICA territory infarct. (C) Approximately 50% of intracranial metastases may be solitary; however, findings in this case argue against a metastatic lesion. (D) Although the lesion presented is hypodense on CT, it is intraaxial and demonstrates restricted diffusion, both of which rule out an arachnoid cyst. (E) Epidermoid cysts can show restricted diffusion, and some can be intra-axial; however, in this case, the vascular distribution is characteristic of a PICA infarct.
A man who was found lying on the ground underwent an emergent CT of the head, shown in these
images. What is the likely diagnosis?
A. Ruptured cerebral aneurysm
B. Acute arterial infarction
C. Global anoxic injury
D. Venous thrombosis
C. Global anoxic injury
This is a case of diffuse cerebral edema/global
anoxic injury. The noncontrast head CT shows
effacement of the cerebral sulci, small ventricles,
loss of the gray matter–white matter distinction,
lack of definition of the deep gray nuclei, and effacement of the basal cisterns. (A) A ruptured intradural cerebral aneurysm would result in subarachnoid
hemorrhage. The hyperdense appearance of the
suprasellar cistern in this case often is referred to
as “pseudosubarachnoid hemorrhage” and is due
to the relative hypodensity of the swollen brain
parenchyma and cisternal effacement. (B) Acute
arterial infarction would lead to edema and mass
effect in a territorial distribution. (D) Venous thrombosis may or may not result in ischemia that sometimes can become hemorrhagic. Findings that can
be seen on CT include a hyperdense clot within
the dilated vein (cord sign), cortical/subcortical
edema (when cortical veins are involved), and deep gray matter edema (when deep draining veins are involved and often occurring bilaterally).
A young woman underwent an MRI study, shown
in these images. What tumor is she at risk of
developing?
A. Glioma
B. Meningioma
C. Ependymoma
D. Endolymphatic sac tumor
E. Subependymal giant cell tumor
A. Glioma
This is a case of neurofibromatosis type 1. The
provided images show dysplasia of the left sphenoid bone with ipsilateral proptosis as well as a
diffuse, nodular, and trans-spatial lesion involving
the left head, face, neck, and parapharyngeal tissues compatible with plexiform neurofibromata. A
central T2 dark spot may be seen within some of
these lesions (the “target sign”) and is considered
highly suggestive of neurofibromas. Approximately
15% of children with neurofibromatosis type 1
develop optic nerve gliomas, and they are also at
an increased risk of developing gliomas in other
regions of the brain. In addition, 4% of patients with
neurofibromatosis type 1 develop peripheral malignant nerve sheath tumors. (B, C) Multiple inherited
schwannomas, meningiomas, and ependymomas
(MISME syndrome) are features of neurofibromatosis type 2 and not type 1. (D) Endolymphatic sac
tumors are very rare neoplasms that can occur
sporadically and have been reported to develop in 10 to 15% of patients with von Hippel–Lindau syndrome. (E) Almost all subependymal giant cell tumors are seen in the setting of tuberous sclerosis.
What factor favors an epidural versus a subdural
empyema?
A. Restricted diffusion
B. Crescentic shape
C. Crosses sutures
D. Less common cerebral edema
E. Peripheral enhancement
D. Less common cerebral edema
Epidural empyemas are associated less frequently
with cerebral edema due to the presence of thick
intervening dura between the collection and the
brain. (A) Both epidural and subdural hematomas
show restricted diffusion. (B) Similar to subdural
hematomas, a crescentic shape is more consistent
with a subdural hematoma, whereas epidural collections characteristically have a biconvex or lenticular shape. Note that subdural empyemas and
hematomas sometimes can show a lenticular shape
particularly if they are loculated. (C) Epidural collections including empyemas and hematomas do
not cross sutures due to the firm dural attachments at these sites. (E) Both subdural and epidural
empyemas enhance peripherally
A mass, shown in this image, was discovered in a
29-year-old woman presenting with progressive
bitemporal hemianopsia and hyperprolactinemia.
Immunohistochemistry and electron microscopy
following resection demonstrated chromophobic
tumor cells and “misplaced exocytosis” with
extrusion of secretory granules. What neuroimaging feature is highly suggestive of cavernous sinus
invasion?
A. Extension beyond the lateral intercarotid line
B. Carotid encasement greater than 180 degrees
C. Obliteration of the superior venous
compartment
D. Obliteration of the inferolateral venous
compartment
A. Extension beyond the lateral intercarotid line
measured on coronal images, is highly suggestive
of cavernous sinus invasion by a pituitary adenoma with a positive predictive value of 85%.
Other findings include obliteration of the carotid
venous sulcus compartment (the space between the sphenoid bone and the ipsilateral cavernous
internal carotid artery) with a positive predictive
value of 95% and internal carotid artery encasement of ≥ 67% (about 240 degrees) with a positive predictive value of 100%. (B) Carotid encasement of ≥ 67% has been associated with the highest likelihood of ipsilateral cavernous sinus invasion. (C) Obliteration of the superior venous compartment is not significantly associated with cavernous sinus invasion. (D) Obliteration of the inferolateral venous compartment is not significantly associated with cavernous sinus invasion.
What is a characteristic of the lesion depicted in
this angiogram?
A. Fibromuscular dysplasia predisposes to an
increased risk of direct-type lesions.
B. Spontaneous intracranial hemorrhage is the
most common presentation.
C. Dural-type lesions commonly present with a
subjective bruit.
D. A majority of direct-type lesions result from
venous thrombosis.
E. Dural-type lesions most commonly present in
young males.
A. Fibromuscular dysplasia predisposes to an
increased risk of direct-type lesions.
The angiogram shows early opacification of the
cavernous sinus compatible with a carotid cavernous fistula. This is most evident on the left side,
where the ophthalmic vein also can be seen. These
lesions result from either a direct communication
between the carotid artery and cavernous sinus
(direct or high-flow fistulas; Barrow type A) or
between the cavernous sinus and dural branches
of the internal carotid artery, external carotid artery, or both (indirect, low flow, and dural fistulas;
Barrow types B, C, and D, respectively). Other than
trauma and aneurysmal rupture, conditions associated with direct carotid cavernous fistulas include
fibromuscular dysplasia, Ehlers-Danlos syndrome,
and pseudoxanthoma elasticum. (B) Spontaneous
parenchymal or subarachnoid hemorrhage can be
seen in direct fistulas, particularly when there is
retrograde cortical venous flow, and occurs in about
5% of cases. Bruits and visual manifestations are much more common. (C) Direct (not dural type) fistulas commonly present with a subjective bruit (about 85% of cases) that sometimes also may be auscultated. (D) Venous thrombosis is a known mechanism in the development of some carotid cavernous fistulas; however, by far the most common etiologies of direct fistulas are trauma and aneurysm rupture. (E) Dural fistulas most commonly present in middle-aged and elderly women. Traumatic direct fistulas are more common in young males.
The tumor depicted in the MRI shown in these
images demonstrated microcysts and mild pleomorphism on histological examination and stained
positive for glial fibrillary acidic protein, neuronspecific enolase, and neuronal cell adhesion molecule. What is true about this lesion?
A. Associated with a high recurrence rate
B. Presents most commonly in young adults
C. Frequently complicated by hemorrhage
D. Associated with TSC-1 and TSC-2 gene
mutations
E. Usually shows minimal to no contrast enhancement
E. Usually shows minimal to no contrast enhancement
Subependymomas are benign, slow-growing
neoplasms classified as WHO grade I. They typically show minimal to no contrast enhancement;
however, this is variable, as some subependymomas may enhance more prominently, particularly
those located in the fourth ventricle. They may
have microcystic changes (both on histological examination and on MRI), are well circumscribed,
are noninvasive, and may contain foci of calcification. The classic location in the fourth ventricle is
at the obex, although they can occur anywhere in
the ventricular system. (A) Subependymomas are
highly resectable neoplasms. Recurrence is rare and
usually related to incomplete excision. (B) Subependymomas present most commonly in middleaged and elderly adults, with a peak incidence in
the fifth and sixth decades of life. Symptomatic
patients may present earlier with signs of intracranial hypertension. (C) Hemorrhage may occur with
subependymomas, but it is a rare event. (D) Subependymomas are not associated with TSC-1 or
TSC-2 mutations, in contrast to the subependymal
giant cell astrocytomas of tuberous sclerosis.
What is a feature of the focal areas of signal intensity (FASI) or unidentified bright objects (UBOs) in
neurofibromatosis type 1?
A. They are most common in basal ganglia and
dentate nuclei.
B. They are premalignant lesions.
C. A small proportion show contrast enhancement.
D. The presence of mass effect is characteristic.
A. They are most common in basal ganglia and
dentate nuclei.
The focal areas of signal intensity (FASI) or
unidentified bright objects (UBOs) are thought to
represent myelin vacuolization in patients with
neurofibromatosis type 1. They are most common
in the basal ganglia, dentate nuclei, and brainstem.
(B–D) These lesions are not premalignant, and
many regress in adulthood. They do not show contrast enhancement or mass effect, the presence of which should raise concern about a glioma.
What MRI sequence specifically should be included
to evaluate a patient with suspected cerebral
abscess?
A. Susceptibility weighted
B. Diffusion weighted
C. Time of flight
D. Constructive interference in steady state (CISS)
E. FLAIR
B. Diffusion weighted
The two most important sequences in the evaluation of a cerebral abscess are contrast-enhanced
T1-weighted sequences and diffusion-weighted
imaging (DWI). An abscess usually shows a relatively smooth and homogeneous enhancing wall
that often (but not always) is thicker on the side of
the gray matter and thinner toward the ventricles
and white matter. Pus within an abscess almost
always shows prominent restricted diffusion,
which is reflected as bright signal on DWI and
corresponding dark signal on apparent diffusion
coefficient (ADC) maps. (A) Susceptibility-weighted imaging (SWI) is a gradient echo–based technique
that is exquisitely sensitive for the detection of
paramagnetic and diamagnetic substances such as
ferritin, calcium, hemosiderin, and deoxyhemoglobin. The capsule of an abscess may appear dark
on SWI or T2, presumably due to the presence of
paramagnetic free radicals within macrophages.
(C) Time of flight is an MR angiography technique
that does not routinely require the administration
of contrast material and is based on the flowrelated enhancement of spins entering an imaging
slice in one direction. (D) Constructive interference in steady state (CISS) is an MRI technique that
is based on consecutive steady-state free precession
(SSFP) sequences. It produces images that have
very high T2 signal and therefore excellent contrast between fluid and parenchyma. (E) Fluidattenuated inversion recovery (FLAIR) is a pulse
sequence that uses an inversion recovery technique to null the signal of simple fluid.
A 74-year-old man underwent a CT study of the
chest, which demonstrated the lesion shown in
these images. This lesion is compatible with what
diagnosis?
A. Metastasis
B. Paget disease
C. Hemangioma
D. Plasmacytoma
C. Hemangioma
These CT images show the characteristic imaging features of a vertebral hemangioma with vertical trabeculations on the sagittal image and a
corresponding “polka dot” appearance on the axial
image. Note the preservation of the cortex despite
the presence of the trabeculations; however, also
remember that hemangiomas can be aggressive
and may present with extraosseous soft tissue
extension and mass effect on the spinal cord or
nerves. On MRI, they usually are bright on both T1
and T2 sequences due to the presence of a fatty
stroma, although atypical (predominantly vascular) hemangiomas may be dark on T1 and simulate
metastases. Approximately 30% of hemangiomas
are multiple. (A) Metastasis would result in variable degrees of marrow replacement and osseous
destruction without the vertical trabecular pattern of a hemangioma. (B) Paget disease is associated with expansion of the bone; in the spine, the
involved vertebra often appears larger than the
ones above and below it. Paget disease also presents with cortical thickening and would not show
a uniform, vertical trabecular pattern. (D) Plasmacytomas cause osseous destruction and do not
respect the trabeculae.
A 67-year-old man with left L5 radiculopathy
underwent MRI of the lumbar spine, shown in
this image. This lesion likely represents a(n):
A. Extruded disk
B. Synovial cyst
C. Ligamentum flavum hypertrophy
D. Uncovertebral joint hypertrophy
B. Synovial cyst
The axial T2 image shows a well-circumscribed,
cystic-appearing, extradural mass, with its base
along a degenerated and hypertrophied left facet
joint, which is compatible with a synovial cyst. In
this case, the cyst impinges on the left descending
nerve roots, which explains the patient’s symptoms. The morphology of the cyst and its anatomic
relationship with a degenerated facet joint are more
important than the signal of its contents, which
may vary depending on the presence of hemorrhage or proteinaceous material. Synovial cysts can
contain gas, and their walls may show enhancement or calcification. They most commonly develop
in patients older than 60 years of age and are more
common in females; 90% occur in the lumbar
spine. (A) In this case, the open base of the lesion is
centered along the facet rather than the disk, and
the T2 bright contents also seem to communicate
with the facet joint. Additionally, the lesion has
a convex, well-circumscribed margin anteriorly.
These features make a synovial cyst the right diagnosis; however, note that disk material can separate from the parent disk and migrate virtually
anywhere within the epidural space. (C) Cysts also
can originate from the ligamenta flava; however,
the lesion in this case is centered along the facet
joint. (D) The uncovertebral joints form between
the uncus and uncinate processes of the cervical
spine between C3 and C7. There are no uncovertebral joints in the lumbar spine. Additionally, hypertrophied bone and osteophytes would look dark on
T2 and not bright as in this lesion.
What is true regarding the lesion indicated by the
arrow in the contrast-enhanced T1-weighted MRI
shown in this image?
A. It is usually angiographically occult.
B. It is composed of radially arranged medullary
veins emptying into a dilated draining vein.
C. High flow from shunting may result in flowrelated aneurysms.
D. A larger nidus is associated with an increased
surgical risk.
B. It is composed of radially arranged medullary
veins emptying into a dilated draining vein.
This image shows radially arranged medullary
veins in a “caput medusae” configuration emptying into a dilated draining vein compatible with a
developmental venous anomaly (also known as a
venous angioma). These lesions usually are benign
and constitute an incidental finding in the majority of cases, although rarely they may thrombose.
Some reports have described seizures, headaches,
and hemorrhage, but the association of venous
angiomas and these manifestations has not been
established firmly. Approximately 20 to 30% of
these lesions are associated with cavernous malformations, which may explain symptoms in at
least some cases. The majority of developmental
venous anomalies are isolated, but they also can
be multiple particularly when associated with the blue rubber bleb nevus syndrome. Blood oxygen
level dependent (BOLD) imaging sequences, such as
susceptibility-weighted imaging (SWI), are the preferred modality for their detection, as these lesions
may be missed on other sequences due to their
slow flow. (A) Developmental venous anomalies
have a characteristic appearance on angiography,
showing exclusive enhancement in the venous
phase, although sometimes a faint blush may be
evident in the late arterial phase. Angiographically
occult vascular lesions include capillary telangiectasias and cavernous malformations. (C) High flow
from shunting may result in flow-related aneurysms in arteriovenous malformations. There is no
arteriovenous shunting in developmental venous
anomalies. (D) A larger nidus is associated with
an increased surgical risk in arteriovenous malformations and upgrades these lesions in the SpetzlerMartin grading system.
A patient presents to clinic for follow-up after suffering from a nontraumatic retinal detachment. The
lesion shown in these images is found on a subsequent MRI. The patient should be screened for:
A. Lisch nodules
B. Bilateral vestibular schwannomas
C. Subependymal giant cell astrocytoma
D. Renal cell carcinoma
E. Low levels of serum ceruloplasmin
D. Renal cell carcinoma
The MRI shows a heterogeneously enhancing
destructive lesion in the left petrous bone. There
are areas of intrinsic T1 hyperintensity secondary
to hemorrhage, proteinaceous contents, or cholesterol. The appearance and location are suggestive
of an endolymphatic sac tumor, particularly in a
patient presenting with spontaneous retinal detachment, which may be seen in von Hippel–Lindau
(VHL) syndrome due to hemorrhage from retinal
capillary hemangioblastomas; 15% of patients with
VHL develop an endolymphatic sac tumor, and 40
to 70% of them ultimately develop renal cell carcinoma, the incidence of which increases with age.
(A) Lisch nodules are the most common ocular
lesions in neurofibromatosis type 1. They are not
associated with VHL syndrome. (B) Bilateral vestibular schwannomas are diagnostic of neurofibromatosis type 2. (C) Subependymal giant cell
astrocytomas occur in patients with tuberous sclerosis complex secondary to mutations in the TSC-1 or TSC-2 genes, although isolated tumors rarely
have been described. These tumors are not associated with VHL. (E) Low levels of serum ceruloplasmin result in an abnormal accumulation of copper
in Wilson disease.
What is a characteristic of the condition depicted
in the T2-weighted MRI shown in this image?
A. Lined by dysplastic white matter
B. Frequently accompanied by microcephaly and other cerebral anomalies
C. Lined by endodermal endothelium
D. May show variable signal intensities depending on its contents
E. Results from a bilateral vascular insult to the
anterior cerebral circulation in utero
B. Frequently accompanied by microcephaly and other cerebral anomalies
This is a case of “open lip” (type 2) schizencephaly, which may result from a variety of insults in utero (including infection and ischemia) that affect the germinal zone during cortical development and lead to abnormal neuronal migration.
There also has been an association with COL4A1
and COL4A2 mutations. Schizencephaly manifests
as a transmantle cleft lined by dysplastic gray
matter (not white matter) that connects the subarachnoid space with the ventricular system (pia
to ependyma). It has been associated with various
anomalies including gray matter heterotopia, polymicrogyria, absence of the septum pellucidum,
and frontal lobe dysplasia. Approximately 80% of
cases are “open lip” and the rest are “closed-lip.”
(A) Schizencephaly is lined by dysplastic gray matter and not white matter, as would be the case in
porencephalic cysts. (C) Schizencephaly is lined by
dysplastic gray matter and not endodermal endothelium, which would be the case in a neurenteric
cyst. (D) A schizencephalic cleft openly communicates with the subarachnoid and intraventricular
spaces, and therefore follows cerebrospinal fluid
signal on all sequences. Cysts with proteinaceous
(e.g., neurenteric cysts) or hemorrhagic contents
may show variable signal intensities. (E) A bilateral
vascular insult to the anterior and middle cerebral
arteries in utero would lead to hydranencephaly
and destruction of parenchyma in these territories with preservation of the posterior circulation; however, large bilateral schizencephaly with hydrocephalus may be difficult to differentiate from hydranencephaly. Some authors suspect that they may be part of a continuum.
A 12-year-old boy with progressive headaches and
lower cranial nerve palsies underwent an MRI
scan, shown in these images. Which is the likely
diagnosis?
A. Chondrosarcoma
B. Chordoma
C. Nasopharyngeal carcinoma
D. Meningioma
E. Lymphoma
B. Chordoma
Skull base chordomas most commonly present
in patients between 20 and 40 years of age, but
16% occur in patients younger than 18 years of age.
Location in the lower clivus with associated osseous erosion is typical. On MRI, they demonstrate
very bright T2 signal, presumably related to mucinous contents along with thin T2 dark fibrous
septations. They show moderate to marked enhancement often along the septations in a “honeycomb” configuration. Skull base chordomas carry a
poor prognosis, worse than that for chondrosarcomas. (A) Chondrosarcomas have imaging features
similar to those for chordomas, but they more often
occur off midline and centered at the petroclival
fissure, and may be accompanied by calcified chondroid matrix. Chondrosarcomas are unusual at this
age. (C) Nasopharyngeal carcinoma is rare at this
age; it would have a more infiltrative appearance,
and it is not as bright on T2 sequences. Additionally,
the nasopharyngeal mucosa is preserved in this
case, and the tumor is centered at the skull base,
resulting in anterior deviation of the prevertebral
muscles as seen on the axial images. (D) Meningiomas can invade bone but tend to do so more diffusely. They also are not as bright on T2 sequences
(unless there are areas of degeneration) and tend
to show more homogeneous enhancement without the honeycomb appearance of this case. They
are rare in children unless associated with a syndrome. (E) Lymphoma can arise from lymphoid
tissue in the nasopharynx and extend to the skull
base; however, the nasopharyngeal mucosa in this
case is preserved. Lymphoma also usually enhances
more avidly and homogeneously.
A man presented with dysfunction of the left cranial nerves V and VI. T1- and T2-weighted MRI and
the corresponding apparent diffusion coefficient
(ADC) map are shown in these images. What is the
patient’s likely diagnosis?
A. Cholesterol granuloma
B. Cholesteatoma
C. Trapped secretions within pneumatized
petrous cells
D. Trigeminal schwannoma
A. Cholesterol granuloma
This is a well-circumscribed, expansile lesion
centered in the left petrous apex. It does not
enhance, and it shows intrinsic T1 brightness as
well as T2 dark material within it, corresponding to
blood products. There is no restricted diffusion as
demonstrated by increased signal on the apparent
diffusion coefficient (ADC) map. Findings are
compatible with a cholesterol granuloma. These
are benign lesions related to repeated middle ear
infections. They contain cholesterol crystals, lipids,
and fluid and are prone to recurrent hemorrhage.
Cholesterol granulomas can occur anywhere in
the temporal bone where there are mucosa-lined
aerated cells, and rarely have been reported in the
paranasal sinuses. (B) Cholesteatomas can occur in
this location but show restricted diffusion (they
would be dark on ADC and bright on diffusionweighted imaging) and usually are not bright on
T1. (C) Trapped secretions within pneumatized
petrous cells sometimes can show intrinsic T1
brightness due to proteinaceous contents, but they
would not result in an expansile mass. The normal trabeculae should be preserved, although this
is difficult to visualize on MRI. (D) A trigeminal
schwannoma would show enhancement and would
follow the course of the cranial nerve sometimes
with involvement of the Meckel cave.
What is the likely mechanism that caused the
injury shown in this image?
A. Lateral bending and compression
B. Axial rotation
C. Abrupt extension
D. Axial loading
E. Flexion and distraction
E. Flexion and distraction
The sagittal CT image shows a compression fracture involving a midthoracic vertebra with a fracture line extending posteriorly across the spinous
process above. This is consistent with a Chance
fracture, which features a horizontal fracture orientation through the entire spinal column and is
a result of a flexion and distraction injury. These
fractures usually are associated with lap belts.
Neurologic injury is rare, but there is an increased
frequency of intra-abdominal injuries. Up to 25%
of Chance fractures may be purely ligamentous.
(A) Extreme lateral bending can result in a lateral
compression fracture or, potentially, a fracture of
the transverse processes. Avulsion fractures of the
transverse processes in the lumbar spine also can
occur due to abrupt contraction of the psoas muscles. (B) Axial rotation can lead to facet dislocations, particularly in the cervical spine. (C) Abrupt
extension of the neck can disrupt the anterior longitudinal ligament and result in an extension teardrop fracture, which is an unstable injury. (D) Axial
loading can lead to burst or compression fractures of the anterior column if the injury occurs during flexion.
A 37-year-old man who is an intravenous drug user
presented with headaches. An MRI of the brain was
performed, shown in these images. What is an
additional expected imaging finding?
A. Presence of lactate and amino acids on MR spectroscopy
B. Hyperintense capsule on T2 sequence
C. Increased apparent diffusion coefficient values
in the center of the lesion
D. High relative cerebral blood volume ratio in
the capsule relative to white matter
A. Presence of lactate and amino acids on MR spectroscopy
The images in this case show an intra-axial lesion
with peripheral enhancement and increased signal
on diffusion-weighted imaging (DWI). Although
apparent diffusion coefficient (ADC) maps are not
presented, the degree of brightness in the center
of the lesion on DWI is high enough that this is
expected to represent restricted diffusion and
would therefore look dark on ADC maps. These
findings essentially are diagnostic of an abscess; a
few rare cases of necrotic metastases with central
restricted diffusion have been reported. Abscesses
can show various amino acids on MR spectroscopy,
including alanine, valine, leucine, and succinate as
well as lactate and lipid peaks. Of note, necrotic primary or metastatic tumors also may show lactate
and lipid peaks. (B) The capsule of a cerebral abscess
usually is iso- to hypointense on T2-weighted
sequences. This is attributed to the presence of
collagen, blood products, or free radicals generated
by phagocytosing macrophages. (C) The necrotic
center of an abscess demonstrates significant
restricted diffusion that manifests as very bright
signal on DWI and low ADC values. (D) The capsule
of an abscess typically has lower relative cerebral blood volume (rCBV) relative to white matter. Glioblastomas, which may look similar to abscesses, have been associated with high rCBV ratios in the capsule.
A 75-year-old man underwent a CT scan of the
head. Representative images are shown here. What
is the patient’s likely diagnosis?
A. Multiple myeloma
B. β-thalassemia
C. Prostate cancer metastases
D. Paget disease
D. Paget disease
The etiology of Paget disease is not entirely certain. Its hallmark is excessive bone remodeling and
overgrowth and is defined by three stages: lytic,
mixed, and sclerotic. The spine and skull are the
most common sites of involvement followed by
the pelvis. The presence of pain or focal destructive lesions on imaging should raise concern for
sarcomatous degeneration. (A) Multiple myeloma
characteristically shows numerous lucent (lytic)
lesions throughout the calvaria that may result in
a “punched out” appearance. (B) Bone marrow
hyperplasia in β-thalassemia can lead to significant skull thickening often showing a so-called
hair-on-end appearance. It would not produce the
lucent and sclerotic pattern of this case. (C) Prostate cancer metastases typically are osteoblastic
(sclerotic) and can be diffuse; however, they do not
result in the mixed lytic/sclerotic geographic pattern of this case. Note on the provided images that,
despite the diffuse involvement of the calvaria, the
inner and outer tables are relatively preserved.
The digital subtraction angiogram shown here
depicts a(n):
A. Middle cerebral artery/anterior cerebral artery
bifurcation aneurysm
B. Posterior communicating artery aneurysm
C. Basilar tip aneurysm
D. Ophthalmic artery aneurysm
B. Posterior communicating artery aneurysm
The image shown is a lateral projection of a
digital subtraction angiogram following a carotid
artery injection. It shows a vascular outpouching
projecting posteriorly from the dorsal aspect of the
C7 (communicating or terminal) segment of the
internal carotid artery at the expected origin of
the posterior communicating artery. Aneurysms in
this location represent about 30% of all ruptured
intracranial aneurysms and may result in intraparenchymal hemorrhage, typically in the mesial
temporal lobe. Patients may present with subarachnoid hemorrhage or a third cranial nerve palsy due
to mass effect. (C) This is a carotid injection and
thus does not depict the posterior circulation and
basilar tip. (D) The ophthalmic artery arises from the C6 (ophthalmic or supraclinoid) segment of the internal carotid artery and projects anteriorly.
A 53-year-old man is status post–glioblastoma
resection and radiation therapy with concomitant
temozolomide. A brain MRI shows an increasing
enhancing lesion. What neuroimaging finding
would favor the presence of recurrent tumor over
radiation necrosis/pseudoprogression?
A. Increased apparent diffusion coefficient (ADC)
values
B. Increased relative cerebral blood volumes
C. “Cut green pepper” appearance
D. Decreased FDG uptake on PET
B. Increased relative cerebral blood volumes
Studies have shown that recurrent tumor has a
higher relative cerebral blood volume (rCBV) normalized to the contralateral white matter compared with radiation necrosis, although there is some
overlap. rCBV values greater than 2.6 have been
suggested as a cutoff for the presence of recurrent
tumor, and values lower than 0.6 suggest pseudoprogression. (A) Several studies show that recurrent tumor tends to have lower apparent diffusion
coefficient (ADC) values compared with radiation
necrosis. ADC ratios related to the contralateral
white matter appear to be more accurate than absolute ADC values. (C) A “cut green pepper,” “soap
bubble,” or “Swiss cheese” appearance has been
described in radiation necrosis, although this usually does not allow reliable discrimination between these processes. (D) Decreased FDG uptake on PET favors radiation necrosis. False negatives may occur in the presence of a large area of necrosis.
A 48-year-old woman with a history of HIV underwent a brain MRI, shown in this image. The MRI
findings are consistent with:
A. Lymphoma
B. Progressive multifocal leukoencephalopathy
C. HIV encephalopathy
D. Cerebritis
C. HIV encephalopathy
HIV encephalopathy is the most common central nervous system infection related to the HIV
virus. Although the incidence of frank dementia
has decreased substantially following the advent
of highly active antiretroviral therapy, the prevalence of mild to moderate cognitive deficits in this
population has increased and probably is at least
partially related to longer survival. Note the prominent cerebral volume loss for the patient’s age,
lack of mass effect, and symmetric confluent white
matter signal abnormalities that spare the U fibers.
(A) Lymphoma would have some degree of mass
effect and would be more focal than the image
demonstrates. Lymphoma in patients with HIV/
AIDS commonly has central necrosis, whereas
lymphoma occurring in the nonimmunocompromised population is diffusely solid and enhances
homogeneously. (B) Progressive multifocal leukoencephalopathy usually is bilateral but asymmetric. When there are subcortical signal abnormalities, they virtually always involve the U fibers; note that these are spared in this case and are seen as dark gray bands between the cortex and the bright white matter lesions. (D) Cerebritis would present with variable degrees of gray and white matter edema.
What MRI sequences are necessary for the evaluation of Hirayama disease?
A. Gradient echo sequences
B. Delayed contrast sequences
C. Diffusion-weighted sequences
D. Flexion/extension sequences
D. Flexion/extension sequences
Flexion/extension sequences are important in the
evaluation of Hirayama disease (monomelic amyotrophy), as they would demonstrate detachment and anterior displacement of the dura on neck
flexion, with resultant spinal cord compression
and myelopathy. Hirayama disease is a benign and self-limiting disease that usually occurs in patients between 15 and 25 years of age. (A) Gradient echo sequences in the spine are useful to differentiate disks from osteophytes and to demonstrate foci of hemorrhage within the cord. These sequences also are less prone to cerebrospinal fluid pulsation artifact. (B) Routine postcontrast sequences help delineate the enhancing epidural space in Hirayama disease and may facilitate identification of the dura. Delayed contrast sequences are not particularly helpful. (C) Diffusion-weighted sequences are technically difficult to acquire in the spine. They may be useful in the evaluation of acute spinal cord ischemia and spinal infection.
A 45-year-old immigrant presents with a several
month history of back pain and malaise. There is
no history of fever. An MRI was performed, shown
in this image. What is the likely diagnosis?
A. Pyogenic diskitis/osteomyelitis
B. Tuberculous spondylitis
C. Degenerative disk disease
D. Osseous metastases
B. Tuberculous spondylitis
Tuberculous spondylitis (also known as Pott
disease) originates in the end plates and typically
spreads in a subligamentous fashion to involve
the adjacent vertebrae, frequently, but not always,
sparing the intervertebral disks, as seen in this case.
The provided contrast-enhanced T1 image shows a
dark lesion (presumably fluid/pus) with peripheral
enhancement. There also is enhancement of the
adjacent vertebra above with preservation of the
intervening disk. Tuberculous spondylitis has a
higher incidence of extensive paraspinal abscess
formation compared with pyogenic infections. Its
onset tends to be insidious and gradual, which not
uncommonly leads to a delayed diagnosis. Constitutional symptoms including fever and weight loss
are seen in less than 40% of cases. (A) Pyogenic
osteomyelitis typically involves the intervertebral
disks. In children, infection is thought to start in
the disks, with secondary involvement of the adjacent vertebrae. In adults, the disease is thought
to start in the end plates, with involvement of the
disks in the majority of cases. (C) Degenerative disk
disease would be centered in the intervertebral
disk spaces and may result in secondary degenerative changes in the adjacent end plates. (D) Osseous
metastases usually enhance and tend to involve
the pedicles and posterior elements. They would
not show the cystic appearance of the larger lesion
seen in the provided image.
The presence of a “swirl” sign is described on a CT
of the head. This finding indicates:
A. Thrombosis
B. Malignant transformation
C. Acute extravasation
D. Abscess formation
C. Acute extravasation
The “swirl” sign is seen on noncontrast head CT
and is defined as areas of low attenuation within
an acute intracranial hematoma. It indicates freshly
extravasated and unclotted blood and is associated
with expansion of the hematoma and a worse
prognosis. (A) Thrombosis can present as a hyperdense clot on noncontrast head CT or a filling
defect following the intravenous administration of
contrast material. (B) Malignant transformation can
occur in various neoplasms. It is not associated
with a “swirl” sign. (D) An abscess presents as a
fluid collection with an enhancing wall and surrounding edema. MRI shows central restricted diffusion. It is not associated with a “swirl” sign.
A coronal noncontrast CT reformat of a 35-yearold woman is shown in this image. What process
is likely to be directly related to the represented
disease?
A. Cytokine release and production of mucopolysaccharides
B. Hematogenous spread of poorly differentiated
adenocarcinoma
C. IgG4-positive lymphoplasmacytic infiltration
D. Monoclonal B-cell population
A. Cytokine release and production of mucopolysaccharides
Thyroid orbitopathy can affect any extraocular
muscle, but the most typical presentation is that of
involvement of the inferior, middle, and superior
rectus muscles, which are markedly enlarged in this
case. Involvement of the lateral rectus muscle almost never occurs in isolation. The pathophysiology
is thought to be related to lymphocytic infiltration
and cytokine release, which stimulates fibroblasts
to produce mucopolysaccharides and, in turn, leads
to soft tissue edema due to a hyperosmolar shift.
(B) The most common primary tumors to metastasize to the orbit are breast, lung, prostate, and melanoma. It would be unusual for orbital metastases
to result in this symmetric appearance of extraocular muscle enlargement. (C) IgG4-related disease
of the orbit is being increasingly recognized as a
cause of “idiopathic” inflammatory pseudotumor.
It can be bilateral and involve any orbital structure,
including the extraocular muscles, lacrimal sac,
and optic nerve sheath complex. (D) Lymphoma
may present as a soft tissue mass frequently related to the lacrimal gland or conjunctiva (MALT lymphoma). Other orbital structures also may be involved.
What is an expected radiographic finding in tension pneumothorax?
A. Ipsilateral diaphragmatic elevation
B. Contralateral increased intercostal spaces
C. “Deep sulcus” sign
D. Mediastinal deviation to the contralateral side
E. Tracheal deviation to the ipsilateral side
D. Mediastinal deviation to the contralateral side
A tension pneumothorax is accompanied by
shift of the mediastinal structures to the contralateral side due to a progressive mass effect. (A) The
diaphragm typically is flattened and depressed on
the side of the tension pneumothorax. (B) The ipsilateral (not contralateral) intercostal spaces increase
secondary to increased intrathoracic volume and
pressure on the side of the tension pneumothorax. (C) The “deep sulcus” sign may be seen in pneumothoraces on supine radiographs, as air collects
basally and anteriorly rather than at the apex in an
upright projection. It does not necessarily reflect
a tension pneumothorax. (E) As with mediastinal structures, the trachea may be deviated to the contralateral side in tension pneumothorax.
A CT of the lumbar spine shows a squared vertebra
with a “picture frame” appearance. This finding is
consistent with what disease process?
A. Paget disease
B. Vertebral hemangioma
C. Lytic metastasis
D. Blastic metastasis
A. Paget disease
Paget disease is a disorder of uncertain etiology
where there is excessive bone remodeling and
overgrowth. It most commonly occurs in the spine,
skull, and pelvis. In the osteosclerotic phase, there
is cortical thickening and coarsening of the trabeculations, which often lead to expansion of the
involved bone. In the spine, the vertebrae become
squared, and a thick, sclerotic margin may be seen,
resulting in the “picture frame” sign. (B) Vertebral
hemangiomas can lead to increased vertical trabeculations that may be seen as dense dots on axial
images. (C) Lytic metastases do not cause a “picture frame” appearance. (D) Blastic metastases can
produce the “ivory vertebra” sign if they involve
the entire vertebral body. Blastic metastases can be seen in various malignancies such as prostate and breast cancer, transitional cell carcinoma, and neuroendocrine tumors
What structure is supplied by the arteries shown
in this image (arrows)?
A. Posterior limb of the internal capsule
B. Anterior limb of the internal capsule
C. Anteromedial caudate nucleus
D. External capsule
E. Thalamus
A. Posterior limb of the internal capsule
The arrows point to the anterior choroidal arteries, which originate between the posterior communicating arteries and the carotid termination.
The posterior communicating arteries can be seen
immediately medial to these on the provided image.
The anterior choroidal arteries supply the posterior
limbs of the internal capsules, lateral geniculate
nuclei, optic tracts and chiasm, hippocampi, amygdalae, and choroid plexus, among other structures.
(B) The anterior limb of the internal capsule (lower
half) is supplied by the recurrent artery of Heubner, which is a branch of the proximal anterior
cerebral artery. The upper half is supplied by the
lateral lenticulostriate arteries. (C) The anteromedial caudate nucleus as well as the anterior portion
of the lentiform nucleus are supplied by the recurrent artery of Heubner. (D) The external capsule
is supplied by the lateral lenticulostriate arteries,
which are branches of the proximal middle cerebral artery and also supply the lateral aspect of the
putamen and upper half of the internal capsule.
(E) The various vascular territories of the thalamus
all are supplied by branches of the posterior cerebral and posterior communicating arteries
A 23-year-old man is brought to the emergency
department following a high-velocity motor vehicle collision. An emergent CT of the cervical spine
is shown in this image. What is a characteristic of
this lesion?
A. Uncommon in children
B. High incidence of neurovascular injury
C. Occurs secondary to axial loading
D. May be identified by a basion-dental interval
greater than 7 mm
B. High incidence of neurovascular injury
The sagittal CT image shows too much space
between the dens and tip of the clivus (basion–
dens interval) consistent with atlanto-occipital
dissociation. This is a severe craniocervical injury
that results from sudden deceleration, with hyperflexion or hyperextension of the head, which disrupts the alar, cruciate, and apical ligaments and
tectorial membrane. The condition is fatal in the
great majority of patients secondary to a high incidence of neurovascular injury. (A) Atlanto-occipital
dissociations are two to three times as common
in children as in adults, presumably due to their
relatively large head sizes, small size of their occipital condyles, and near horizontal orientation of
the atlanto-occipital joints. (C) Other less severe
injuries such as the Jefferson fracture may occur
secondary to axial loading. (D) Atlanto-occipital
dissociation injuries may be identified by a basion–
dens interval greater than 12 mm as originally
measured on lateral cervical spine radiographs. One study suggest a lower cutoff value (8.5 mm) with CT imaging, due to the presence of magnification on plain radiography.
A 45-year-old woman with history of breast cancer
underwent an MRI study of the brain. What is a
common clinical manifestation of the process presented in these images?
A. Meningismus
B. Headache
C. Ischemia
D. Diabetes insipidus
E. Dysarthria
B. Headache
Leptomeningeal carcinomatosis occurs in about
5% of patients with metastatic cancer and portends
a poor prognosis. Half of these patients present
with headaches, which represent the most common clinical manifestation. The incidence of leptomeningeal carcinomatosis is four times higher
in autopsy studies. (A) Meningismus is present in
13% of patients with leptomeningeal metastases.
(C) Ischemia is an uncommon complication of
leptomeningeal carcinomatosis. In contrast, tuberculosis can manifest with leptomeningeal disease
that characteristically results in cerebral infarcts.
(D) Diabetes insipidus can occur in patients with
infundibular involvement but is much less common than headache. It also is more frequently seen
in leukemic patients rather than in patients with
solid tumor leptomeningeal disease. (E) Dysarthria
is an uncommon complication that occurs in 7% of
patients with leptomeningeal metastases.
Axial noncontrast CT and T2-weighted MRI are
shown in these images. What is the likely diagnosis?
A. Langerhans cell histiocytosis
B. Epidermoid
C. Abscess
D. Leptomeningeal cyst
D. Leptomeningeal cyst
The CT shows a cyst-like structure projecting
through a calvarial defect. The T2 image better demonstrates that there is encephalomalacia in area of the brain, and therefore findings are compatible with a leptomeningeal cyst. These findings
develop from fractures with associated dural tears
and herniation of the pia-arachnoid through the
dural defect, with progressive osseous erosion (also
known as a “growing” skull fracture) due to cerebrospinal fluid pulsations. (A) Langerhans cell histiocytosis is an important cause of a lytic calvarial
defect in a child. Given the underlying encephalomalacia demonstrated, findings are more likely
related to prior trauma. (B) An epidermoid can be primarily calvarial, but there is no evidence of it on the MRI shown in these images, which instead shows underlying encephalomalacia. (C) The T2 image demonstrates encephalomalacia and no evidence of an abscess or parenchymal edema.
Additionally, an abscess would not result in a calvarial defect.
A 48-year-old man underwent CT and MR studies
after presenting with left-sided ophthalmoplegia.
The lesion enhanced following the administration
of intravenous contrast material and did not show
restricted diffusion. Representative images are
shown here. What is correct regarding this entity?
A. Commonly produces a chondroid matrix
B. Consists of granulation tissue and cholesterol
crystals
C. Represents trapped secretions within the
petrous apex
D. Arises from aberrant ectoderm within petrous
apex cells
E. Commonly demonstrates low apparent diffusion coefficient values
A. Commonly produces a chondroid matrix
The CT image shows a destructive lesion involving the left petrous apex and petroclival junction,
with contrast enhancement on the MRI study. There
are small, calcific densities within it that could
represent either chondroid matrix or eroded bone.
Of the provided options, this lesion is most consistent with a chondrosarcoma, which comprises a
heterogeneous group of malignant neoplasms characterized by the production of a chondroid matrix
(sometimes seen as “rings” and “arcs” within a
lytic lesion). About 1% of chondrosarcomas occurs
at the skull base and commonly slightly off midline at synchondroses such as the petroclival and
spheno-occipital sutures. (B) Cholesterol granulomas are thought to arise secondary to repeated
episodes of hemorrhage within mucosa-lined air
cells. They consist of granulation tissue and cholesterol crystals and typically are hyperintense on
noncontrast T1-weighted images with areas of T2
hypointensity related to blood products. (C) In
trapped secretions, the air cell septations are preserved, and there would not be erosive changes or
an expansile enhancing lesion. (D) Aberrant ectoderm within petrous apex cells may result in a
cholesteatoma, which does not enhance and usually is very bright on diffusion-weighted sequences
due to a combination of restricted diffusion and
T2 shine-through effects. (E) The vast majority of
chondrosarcomas are slow-growing WHO grade I
or II neoplasms that do not show significant
restricted diffusion.
A 42-year-old man with back pain underwent an
MRI examination of the lumbar spine, shown in
this image. What is an expected imaging finding
in this patient?
A. Expansion of the L5 vertebra
B. Sclerosis of the contralateral pars interarticularis
C. Widening of the spinal canal on a midsagittal image
D. Widening of the neural foramina
C. Widening of the spinal canal on a midsagittal image
Anterior listhesis secondary to pars defects (but
not due to facet hypertrophy) often results in widening of the spinal canal on a midsagittal image.
(A) Small size or hypoplasia of the L5 vertebra may
predict the presence of bilateral pars interarticularis defects and should prompt a search for them.
(B) Sclerosis of the contralateral pars interarticularis can occur with a chronic unilateral pars interarticularis defect. This patient likely has bilateral
defects as indicated by significant listhesis of L5
over S1. (D) There may be variable degrees of neural foraminal narrowing depending on the severity of the listhesis, as can be seen on this image.